Skip to main content

Stack Exchange Network

Stack Exchange network consists of 183 Q&A communities including Stack Overflow, the largest, most trusted online community for developers to learn, share their knowledge, and build their careers.

Visit Stack Exchange
Asked
Viewed 488 times
0
$\begingroup$

The problem is following:

  • Suppose we have a line segment with a length $L$ and we randomly choose (with uniform distribution) $n$ points on this segment, so we divide the main line segment into $n+1$ sub-segments.
  • Now we define a random variable which describes length of one of this sub-segment and we want to find it's distribution.
  • My approach to this problem was to firstly construct a random variable $X_i$ which realisations are points on a line segment, we know that it has uniform distribution:

$$ \rho_{X_i}(x) = 1/L,\;\; \text{if}\;\; 0\leq x\leq L\;\;\text{and}\;\; \rho_{X_i} = 0\;\;\text{elsewhere.} $$

  • Then we define a random variable $$ D_{i} = X_i - X_{i-1}\quad\mbox{with conditions that}\quad X_0 = 0,\ X_{n+1} = L $$ So we want to find distribution of a random variable $D_i$.
  • After short inspection of “Convolution of Probability”, I found out that distribution of SUM of the independent random variables is a convolution of distributions, but I don't know what is a formula for a difference.
  • My first guess was that it is a convolution but with a plus sign: $\int dy \rho (x+y)\rho (y)$ (which in a case of uniform distributions doesn't change anything), but the result is wrong - convolution of two constant functions is a constant function - and I expect something different (maybe an exponential distribution, idk).

I will be thankful for any advice.

$\endgroup$
3
  • 1
    $\begingroup$ The first issue with your approach is that there is no reason why $X_i - X_{i-1}$ must be positive. It's true that the pdf of each $X_i$ is $1/L$, but there is no ordering of these variables. What you might want to use is the order statistics, $X_{(i)}$ -- so for example, $X_{(1)} \equiv \min \{ X_i \}$. These have their own pdfs that will differ according to $i$. Have you worked with those before? $\endgroup$
    dmk
    –  dmk
    2024-05-08 12:44:29 +00:00
    Commented May 8, 2024 at 12:44
  • 1
    $\begingroup$ This ought to give you almost everything you need: en.wikipedia.org/wiki/… $\endgroup$
    dmk
    –  dmk
    2024-05-08 13:47:20 +00:00
    Commented May 8, 2024 at 13:47
  • $\begingroup$ Thanks for a hint. I have never worked with order statistics, in this article there is a ready formula for distribution of a random variable of $r$ order. I suppose after using this formula next steps are: $D_i = X_{(i)}-X_{(i-1)}$ and finding $\rho_{D_i}$ by applying convolution? Also I'm looking for some derivations of this formula, because I can't see it on wiki. $\endgroup$
    XaveryXavier
    –  XaveryXavier
    2024-05-08 15:48:17 +00:00
    Commented May 8, 2024 at 15:48

1 Answer 1

1
$\begingroup$

The key bit from the Wikipedia article I mentioned above is its characterization of the difference of two order statistics from a $U(0,1)$ distribution: If $U_1, U_2, \ldots U_n \overset{\text{iid}}{\sim} U(0,1)$, then the difference between order statistics $U_{(k)} - U_{(j)} \sim \text{Beta}(k-j, n-(k-j)+1)$, where $k > j$.

So let's set $L=1$ for the moment. Since you are interested in consecutive order statistics, you'll have $D_i = X_{(i)} -X_{(i-1)} \sim \text{Beta}(1, n)$, for $2 \le i \le n$. If $i = 1$, we have

$$D_1 = X_{(1)} - X_{(0)} = X_{(1)} - 0 = \min\{X_i\},$$

and if $i = n+1$, we have

$$D_{n+1} = X_{(n+1)} - X_{(n)} = 1 - X_{(n)} = 1 - \max\{X_i\}. $$

Perhaps it's not too much of a surprise, given the symmetry of the $U(0,1)$ distribution, that these two edge cases actually have the same distribution. Therefore, we can manually compute the distribution of $\min\{X_i\}$ using straightforward methods:

\begin{align} P(D_1 \le d) &= P(\min\{X_i\} \le d) \\ &= P(\cup[X_i\le d]) \\ &= P(\cup [X_i>d]^\prime) = P([\cap (X_i > d)]^\prime) = 1 - P(\cap[X_i > d]) \\ &\overset{\text{iid}}{=} 1 - \Pi P(X_i > d) = 1 - \Pi [1 - F_X(d)] \\ &= 1 - (1 - d)^n \\ \end{align}

Since $F_{D_1}(d) = 1 - (1 - d)^n$, we have the pdf $f_{D_1}(d) = n(1-d)^{n-1}$; this is the pdf of a $\text{Beta}(1,n)$ variable. (It's not difficult to show that $D_{n+1}$ has the same distribution.)

Therefore, all of your $D_i$ have the same distribution: $\text{Beta}(1,n)$.

Wait! I just remembered: I set $L = 1$ :). Hopefully it's intuitive that your more generally defined $D_i$ is a simple transformation of $\text{Beta}(1,n)$ -- namely, if $W \sim \text{Beta}(1,n)$, then $ D_i = LW$ solves your problem :).

...

So. If you have the result from the Wikipedia page I cited, the problem becomes fairly straightforward. If you don't...

I'm pretty rusty on this stuff. I have some idea of a path through this problem; in fact, I started down one such path before I stumbled upon that very nice result on Wikipedia. But it might take a while to put those thoughts in order. I'll see what I can do, but hopefully the above is satisfactory for the moment :).

I would be hesitant to try convolution because, if you want to try something like $X - Y = X + (-Y)$, then you're gonna have to start defining new random variables, which will have new pdfs and new supports. So it probably won't be as nice as you expect -- regardless of how nice that is :).

$\endgroup$

You must log in to answer this question.

Start asking to get answers

Find the answer to your question by asking.

Ask question

Explore related questions

See similar questions with these tags.

Morty Proxy This is a proxified and sanitized view of the page, visit original site.